Answer:
[tex]\mathrm{Center\:\:}= (- 3,17)\\\mathrm{Radius\:\:}= 13[/tex]
Step-by-step explanation:
Remember that the center of the circle would be equidistant from the given points that lie on the circle.
By determining the diameter through the distance of the given points, we can determine the radius by dividing this distance by 2. And by determining the radius we can determine the center of the circle. Let's do it.
[tex]\mathrm{Distance\:between\:}\left(2,\:5\right)\mathrm{\:and\:}\left(-8,\:29\right) = \sqrt{\left(-8-2\right)^2+\left(29-5\right)^2} = 26[/tex]
[tex]\mathrm{Radius\:\:}= 26 / 2 =13[/tex]
So now that we know the radius is 13 units, we can eliminate answer choice(s) # 2 and # 4. That leaves us with the first and third answer choices. The center being (5,12) is not likely, as most of the circle lies in the 2nd quadrant. That is presumably why (- 3,17) is more predictable. Our answer is the third option.
hose fills a hot tub at a rate of 4.09 gallons per minute. How many hours will it take to fill a 250-gallon hot tub?
Answer:
Step-by-step explanation:
250 gals divided by 4.09
answer:61.12
11) 19, 13, 7, 1...
Explicit:
Recursive:
13) -3,-1,1, 3...
Explicit:
Recursive:
Can someone help me die in 10 min
Answer:happy
Step-by-step explanation:
Vg
this expression 3x+8+4x+2x has four what?
Answer:
polynomials
Step-by-step explanation:
Answer: The expression has four terms.
In Algebra, a term is either a single number or variable, or numbers and variables multiplied together. Terms are separated by + or − signs, or sometimes by division signs.
Hope this helps!
If x+3y=7 and x-3y=1 what's y
Subtract one equation from the other to eliminate x :
(x + 3y) - (x - 3y) = 7 - 1
6y = 6
y = 1
it's from Khan Academy and there are answer choices it's called interpet negative number addition and subtraction expression
Answer: Hi!
Answers B and D are correct.
For B, since negative numbers are on the left side of the number line, a positive number would have to be to the right of a negative number. Since we are adding a negative and a positive number of the same absolute value (9), they would cancel out to 0, which makes D true.
Hope this helps!
Hi there! Hopefully this helps!
-----------------------------------------------------------------------------------------------------
Answers: D, B.~~~~~~~~~~~~~~~~~~~~~~~~~~~~~~~~~~~~~~~~~~~~~~~~~~~~~~Explanation for D: -9 + 9 = 0.
Explanation for B: Negative numbers are on the LEFT side of the number line.
And positive numbers are on the right.
Therefore, 9 is to the RIGHT of -9 on the numberline.
what is 7500×5643 ÷3
Answer:
14107500Step-by-step explanation:
[tex]7500 \times 5643 \\ = \frac{42322500}{3} \\ = 14107500[/tex]
Answer:
[tex]\Large \boxed{14107500}[/tex]
Step-by-step explanation:
There are different ways to do this;
1. ⇒ (7500×5643)÷3
⇒ 42322500÷3
⇒ = 14107500
2. ⇒ 7500*5643/3
⇒ 42322500/3
⇒ =14107500
^ Two ways you could have solved the Q.
Answer; 14107500
Set up an equation for the following scenario: A wedding reception needs 3 chairs for every 4 people.
How many chairs, y, would be needed for x people?
whar is the LCM of 1/6 and 3/8
==========================================
There are 3 methods to find the lowest common multiple (LCM)
---------------------------
Method 1
List out the multiples of 6 and 8
multiples of 6 = {6, 12, 18, 24, 30, 36, 42, 48, ...}
multiples of 8 = {8, 16, 24, 32, 40, 48, ...}
We see that 24 is the smallest number in both sets at the same time. So this is the LCM.
---------------------------
Method 2
Write the prime factorization of 6 and 8
prime factorization of 6 = 2*3
prime factorization of 8 = 2*2*2
The unique prime factors here are 2 and 3. We have 2 show up at most three times meaning that 2*2*2 = 8 is a factor of the LCM. The number 3 is also a factor meaning that 8*3 = 24 is the LCM.
---------------------------
Method 3
Multiply the denominators 6 and 8 to get 6*8 = 48
48 is a common multiple, but its not the smallest
we can divide 48 over the GCF 2 to get 48/2 = 24, which is the actual LCM.
So when you have 2 different denominators, you can multiply their values and then divide by the GCF to get the LCM.
When planning the new town hall, the architect planned for 50 offices. only 5 will be on the 1st floor. what percentages of offices will be on floors above the first floor?
A)5%
B)90%
C)50%
D)10%
Answer:
5 upon 50
5/50 X 100%
1/10 X 100%
10 %
is your answer
plz brainlist me please
What is the slope of the line that passes through the points (-7, - 7) and (-7,1)?
Write your answer in simplest form.
Answer:
Undefined
Step-by-step explanation:
Slope of the line
[tex] = \frac{1 - ( - 7)}{ - 7 - ( - 7)} = \frac{1 + 7}{ - 7 + 7} = \frac{8}{0} = \infty \\ = undefined [/tex]
Which of the following is an example of the associative property of multiplication?
7 · (3 · 5) = (7 · 3) · 5
7 · (3 + 5) = 21 + 35
7 · (3 · 5) = 7 · (5 · 3)
7 · 1/7 = 1
Answer:
Option A
Step-by-step explanation:
7 * (3 * 5) = (7*3) * 5
Associative Property:
When three or more numbers are multiplied, the product is always same regardless of the regrouping
Answer:
a is correct cause foot
-3/4 x (-10/9)
A. 27/40
B. 5/6
C. -27/40
D. -5/6
Answer: B
Step-by-step explanation:
1. Multiplying two negatives equals a positive.
2. Reduce the numbers with the GCF, 3.
3. Reduce the numbers with the greatest common factor, 2.
4. Multiply the fractions which would give you: 5/6
Answer:
B. 5/6
Step-by-step explanation:
1.multiply -3 by -10 (negative multiplying negative is positive) which will give you 30
2.Then multiply 4 by 9 = 36.
3.u will get 30/36, divide both the numerator and denominator by 6.
4. it will give you 5/6 as an answer.
3x + 5 = 2x + 2 -(x-3) Pls show work too, thanks :)
Answer:
X=0
Step-by-step explanation:
1.Remove the parentheses
3x + 5 = 2x + 2 -(x-3) -> 3x+5=2x+2-x+3
2.Combine like terms
3x+5=2x+2-x+3 -> 3x+5=x+5
3.Cancel Equal Terms
3x+5=x+5 -> (subtract 5 from both sides) 3x=x
(subtract x from both sides) 3x=x -> 3x-x=0
4.Combine like terms
3x-x=0 -> 2x=0
5.Get rid 2 so divide both sides by 2
2x= 0 -> [tex]\frac{2x}{2}[/tex]=[tex]\frac{0}{2}[/tex] ->
x=0
Identify the range of the function. A) (2, 8) B) [2, 8] C) (−5, 9) D) [−5, 9]
Answer:
The answer is "Option A"
Step-by-step explanation:
The domain is the collection of the value, which belongs to the separate variable (horizontal axis). So, to find a region with a graph, it must search for the function, which starts and end. And at all these levels we are searching at x-values.
Its starting point is (2,9) and the ending point is (8,3). Therefore, x= 2 to x=8 is the domain.
If ∠P measures 27°, ∠R measures 135°, and p equals 9.5, then which length can be found using the Law of Sines?
Answer:
The length r can be found using the Law of sines.
Length r = 14.8
Step-by-step explanation:
The length r can be found using the Law of sines.
The formula for the Law of Sines states that:
a/Sin A = b/ Sin B
In the above question,
∠P = 27°
∠R = 135°
p = 9.5
r = ????
The formula =
p/Sin P = r/Sin R
9.5/Sin 27° = r/Sin 135°
Cross Multiply
r × Sin 27° = 9.5 × Sin 135°
r = 9.5 × Sin 135°/Sin 27°
r = 14.7966
Approximately Length r = 14.8
What is the formula for the following geometric sequence?
3, 12, 48, 192, ...
Answer:
Step-by-step explanation:
a n = a 1 r n − 1
Find an equation in standard form of the line containing the point (-3, -2) and having slope 14.
Hint: You are given a point and a slope, so start with point-slope form. Then, convert the point-
slope form to standard form by moving all the x and y terms to the left side of the equation.
Answer:
y - 14(x) = 40 (Hopefully)
Step-by-step explanation:
-2 = 14(-3) +40
y = 14(x) + 40
y - 14(x) = 40
Use the distributive property to solve 2(3x+4p-8)
Answer:
6x + 8p - 16
Step-by-step explanation:
Given
2(3x + 4p - 8) ← multiply each term in the parenthesis by 2
= 6x + 8p - 16
Answer:
Step-by-step explanation:
2(3x + 4p - 8)= 2*3x + 2* 4p + 2*(-8)
= 6x + 8p - 16
8(-b+4)=
idk what the answer is please help
Answer:
-8b+36 is the answer.
Step-by-step explanation:
=8(-b+4)
Opening brackets to simplify
=8✖️(-b)+8✖️4
=-8b+36 is the answer.
Hope this will help you :)
please show your work :)
-2x + 4 = 12
Answer:
[tex]\large\boxed{x = -4}[/tex]
Step-by-step explanation:
-2x + 4 = 12
Subtract 4 from both sides of the equation
-2x = 8
Divide both sides of the equation by 2
-x = 4
Transfer the minus (multiply both sides by -1)
[tex]\large\boxed{x = -4}[/tex]
Hope this helps :)
Answer:
x = -4
Step-by-step explanation:
-2x + 4 = 12
-2x = 8
x = -4
1. Find (-9) +7 –(-3)
(-9)+7-(-3)
-9+7+3
-9+10
1
Answer:
1
Step-by-step explanation:
(-9) +7 is -2 because the brackets here don't change anything
so we have -2 - (-3) two negatives make a positive so - (-3) is basically +3 leaving -2+3 so the answer is 1
Hope this helps!
Determine the volume of the CD box shown
7.5 in
6.25 in.
8 in
I hope this helps you
Rectangular Prism Volume is
Height. Width. Height
(6.25).(7.5).(8)
375
A basket of fruit contains 4 bananas, 3 apples, and 5 oranges. You intend to
draw a piece of fruit from the basket, keep it, draw a 2nd piece of fruit from the
basket, keep that, and then draw a third piece of fruit from the same basket. What
is the probability of selecting one banana, then one apple, then one orange, in
that order, from the basket if you are blindfolded?
Hi
draw 1 : 4/12 = 1/3
draw2 : 3/11
draw 3 : 5/10 = 1/2
so probability to have : 1 banana then 1 apple then 1 orange is : 1/3*3/11*1/2 = 3/66 = 3*1 / 2*3*11 = 1/22
Answer:
1/22
Step-by-step explanation:
Work out the probability of selecting each fruit then times them together so
there are 12 fruits in total and 4 bananas so the chance of selecting a banana is 4/12 simplified 1/3
then as you dont put the banana back there is only 11 fruits left in the basket and 3 are apples so the chance of selecting an apple is 3/11
then as you dont put the apple back there is only 10 fruit left in the basket and 5 are oranges so the chance of selecting an orange is 5/10 simplified 1/2
then we multiply them all together 1/3 x 3/11 x 1/2 3/66 to get which simplifies to 1/22
The sum of 2 consecutive whole numbers is 206. What are the 2 numbers
Answer:
102.5, 103.5
Step-by-step explanation:
a+a+1=206
2a+1=206
2a=205
a=102.5
103.5
I think there is a bug here. They can't be whole numbers.
Answer:
There are no two consecutive whole numbers that sum to 206 (See proof below)
Step-by-step explanation:
let the first number be x
since the numbers are whole and consecutive, the second number must be (x + 1)
we are given that the number sum to 206
hence,
x + (x+1) = 206
x + x + 1 = 206
2x + 1 = 206 (subtract 1 from both sides)
2x = 206 - 1
2x = 205 (divide both sides by 2)
x = 205/2 = 102.5 (NOT A WHOLE NUMBER)
if x = 102.5, then the second number must be
x + 1 = 102.5 + 1 + 103.5 (ALSO NOT A WHOLE NUMBER)
Rita is 8 years older than Fati. If Fati is 15 years now, how old was Rita 3 years ago
Answer:
Rita is 20 years old
Step-by-step explanation:
If you add 8 years to Fati's age, (15+8), you get 23. That is Rita's age right now. Now subtract 3 from 23. You get 20. So Rita is 20 years old.
A coat originally cost $130. Now it is on sale for 60% of its original cost. What is the sale
price of the coat?
OA) $78
B) $70
OC) $52
OD) $40
Answer:
Step-by-step explanation:
b-78
∠A and \angle B∠B are supplementary angles. If m\angle A=(7x+10)^{\circ}∠A=(7x+10) ∘ and m\angle B=(7x-26)^{\circ}∠B=(7x−26) ∘ , then find the measure of \angle A∠A
Answer:
x = 25
Step-by-step explanation:
The measure of angle A is 108°
What is a angle?An angle is formed when two straight lines or rays meet at a common endpoint.
What is a supplementary angle?Supplementary angles are those angles that sum up to 180 degrees.
Given that angle A and angle B are supplementary angle
∠A=(7x+10)°
∠B=(7x−26)°
(7x+10)° + (7x−26)° =180°
14x - 16° =180°
14x =196°
x= 14°
∠A=(7x+10)°
∠A=(7(14) +10)°
∠A= 108°
Thus the measure of angle A is 108°
To know more about supplementary angles click here
https://brainly.com/question/13045673
#SPJ2
In her last four basketball games, Alexi has scored 12, 14, 26, and 40 points. If the pattern continues, how many points will Alexi score in her next game?
Answer:
its 26. Just take my word for it
Step-by-step explanation:
14-12=2
26-14=12
40-26=14
You see the pattern. Alexi will score 26 pts. btw, this goes against the laws of probability, but whatever.
It’s either D or E; please help
Thirty overweight males agree to participate in a study of the effectiveness of three weight-loss treatments: X, Y, and Z. The researcher calculates the
body fat composition of each subject. The response variable is the reduction in body fat after six weeks of treatment. Previous studies have shown the
effects of a diet may vary based on a subject's initial body fat composition. How should the subjects be blocked? Explain.
They should be blocked alphabetically so the subjects are as different as possible.
They should be blocked by age because weight loss is more difficult to achieve as you get older.
They should be blocked by height because weight loss is easier to achieve for taller people.
They should be blocked by body fat composition so the subjects are as similar as possible.
Blocking should not be used in this experiment.
Answer:
They should be blocked by body fat composition so the subjects are as similar as possible.
Step-by-step explanation:
It's D .
Blocking here means how should the subjects be arranged in terms of body fat or weight.
Option A is wrong because alphabetically controlling body fat would not get desired results.
Option B is wrong because different age groups would yield different results.
Option C is wrong also because weight loss is not dependent on taller people.
Option D is correct because controlling body fat would yield best possible results in ascending or descending order
Option E is incorrect because arranging data in ascending or descending order would simplify the researches experiments.
Ben rents a car for one day. The charge is $20 plus the $0.12 per mile. He wants to spend exactly $80. How many miles can he drive? Write and solve an equation to find how many miles Ben can driv
Answer:
Ben can drive 500 miles for $ 80
Step-by-step explanation:
Let y be the cost of traveling and x be the number of miles . The charge is $20 plus the $0.12 per mile which means the cost $ 20 is constant and an additional cost of $ 0.12 is spent on every mile. Then according to the given conditions the equation will be
y = $20 + $ 0.12(x) where x is the number of miles
Now we can solve x for y= 80
$ 80= $20 + 0.12 (x)
$ 60 = 0.12x
x= 60/ 0.12= 500
Ben can drive 500 miles for $ 80